Question
In the question given below, a passage/statement is
given followed by three statements which may or may not strengthen/weaken the assertion made in the passage. Answer the question as per the individual direction given. Passage: India’s mid-day meal scheme, which provides free lunches to school children on working days, has been credited with improving nutritional levels and increasing school attendance, especially in rural areas. The scheme has also contributed to reducing caste-based discrimination as children from different backgrounds sit together to eat. Critics, however, point to irregularities in quality and delays in implementation in certain regions. Which of the following, if true, would strengthen the argument in favor of the scheme’s positive impact? I. In many villages, parents cite the mid-day meal as one of the primary reasons for sending their children to school. II. Independent studies have shown an increase in the Body Mass Index (BMI) of children in schools covered under the scheme. III. In some districts, there have been complaints about poor food quality and unhygienic kitchens.Solution
We need to identify statements that support the argument that the scheme is beneficial. Statement I: Yes, it strengthens the argument by linking school attendance to the meal scheme. Statement II: It shows a positive health impact, directly supporting the scheme’s nutritional goals. Statement III: This weakens the argument by pointing to implementation flaws, so it doesn’t strengthen the case.
Read the given statements and conclusions carefully. Assuming that the information given in the statements is true, even if it appears to be at varianc...
If digit 2 is interchanged with digit 1, then what will be the value of the following equation?
9 + 6 ÷ 4 × 21 – 5=?
Which figure from the given options would replace the question mark (?) if the following figure series were to be continued?
Identify the figure that completes the pattern (rotation is NOT allowed).
Select the correct mirror image of the given figure when the mirror is placed at line MN as shown.
Statements: Some lipsticks are foundation. All foundation is brown. Some brown are highlighters.
Conclusion I: Some lipsticks are brown.
...A question is given followed by two arguments. Decide which of the arguments is/are strong with respect to the question.
Question:
Shoul...
In the following question, select the figure which can be placed at the sign of question mark (?) from the given alternatives.
Which two mathematical signs should be interchanged to make the given equation correct?
15 × 3 ÷ 15 – (45 × 3) – 10 + (13 ÷ 2) = 76
...Two positions of a dice are shown below: Which digit will appear to the opposite side face to